Re: [obm-l] Problemas da IMO

2003-07-15 Por tôpico Marcio Afonso A. Cohen
Eu sei que ninguem gosta muito disso, mas esse problema 4 (que eu ateh
imagino que nao seja dificil por plana) eh bem simples na conta bruta.. Eh
impressionante como complexos ajudam nos problemas de geometria da imo..
aquele artigo da eureka 6 eh realmente muito util!

Coloque o circuncentro na origem, e represente os vertices pelos
complexos a,b,c,d, todos de modulo 1u.m.
Reta ab: z+abz' = a+b
Reta perpendicular a ab passando por d: z-abz'=d-abd'
Logo, o ponto P eh 2p = [a+b+d-ab/d]
Portanto, 2q = [a+c+d-ac/d] e 2r = [b+c+d-bc/d].
Como p,q,r sao colineares (reta de simpson), e |p-q| = |q-r|:
p-q = q-r, ou seja: b-c + ac/d - ab/d = a-b +bc/d-ac/d
Arrumando: (b-c) - (a/d)(b-c) = (a-b) - (c/d)(a-b) sse (b-c)(d-a)=(a-b)(d-c)
Tirando modulo, isso significa que BC*AD = AB*DC. E isso fecha o problema.
De fato, sendo I o peh da bissetriz de ABC em AC, entao, AI/IC = AB/BC e vc
quer provar que I eh peh da bissetriz de ADC, i.e, que AI/IC=AD/DC (teorema
da bissetriz interna, ida e volta). Portanto, eh suficiente provar que AB*DC
= AD*BC.

Vou pensar nos outros agora, esse foi o que eu achei que seria mais
facil.. (ja pensei no 2 e no 1 um pouco tmb..)


- Original Message -
From: [EMAIL PROTECTED]
To: [EMAIL PROTECTED]
Cc: [EMAIL PROTECTED]; [EMAIL PROTECTED]
Sent: Monday, July 14, 2003 3:38 PM
Subject: [obm-l] Problemas da IMO




 Prova da IMO retirada do Site http://www.mathlinks.go.ro/

 O Problema 1 é nois que mandou...


 First Day - 44th IMO 2003 Japan

 1. Let A be a 101-element subset of the set S={1,2,3,...,100}. Prove
that
 there exist numbers t_1, t_2, ..., t_{100} in S such that the sets

 Aj = { x + tj | x is in A } for each j = 1, 2, ..., 100

 are pairwise disjoint.


 2. Find all pairs of positive integers (a,b) such that the number

 a^2 / ( 2ab^2-b^3+1) is also a positive integer.

 3. Given is a convex hexagon with the property that the segment connecting
the
 middle points of each pair of opposite sides in the hexagon is  sqrt(3) /
2
 times the sum of those sides' sum.

 Prove that the hexagon has all its angles equal to 120.


 Second Day - 44th IMO 2003 Japan

 4. Given is a cyclic quadrilateral ABCD and let P, Q, R be feet of the
 altitudes from D to AB, BC and CA respectively. Prove that if PR = RQ then
the
 interior angle bisectors of the angles  ABC and  ADC are concurrent on
AC.

 5. Let x1 = x2 = ... = xn be real numbers, n2.

 a) Prove the following inequality:

 (sum  ni,j=1 | xi - xj | ) 2 = 2/3 ( n^2 - 1 )sum ni,j=1 ( xi - xj)^2

 b) Prove that the equality in the inequality above is obtained if and only
if
 the sequence (xk) is an arithemetical progression.

 6. Prove that for each given prime p there exists a prime q such that
n^p - p
 is not divisible by q for each positive integer n.



=
Instruções para entrar na lista, sair da lista e usar a lista em
http://www.mat.puc-rio.br/~nicolau/olimp/obm-l.html
=


[obm-l] Seu crêisson!!! O_Erro_Comum...

2003-07-15 Por tôpico MuriloRFL



Seu sinhoJP,

Nois, da 
faucudadiseu Crêisson R$ ®,convidiamos vc pra nossia faucudadi 
Nois temo aula especialica pra vc... Pra alunio reclamonico e griladiu!!! Temo 
terapia de chókioA faucudade sónecessitia di sua assinaturia 
Provanois num faiz pq nois num sabe mesmiu! nois queremio éseu 
dinheirioE nem precisia escrivinha bestieiria!!! Vc fiquia bem longi 
du lapisu.

  
Gratilissimio,

Faucudadi 
seuCrêisson R$ ® 

Agoria cum brilhiu nus olhius! 
$-;-$
 
X


- Original Message - 
From: J.Paulo roxer 
´til the end 
To: [EMAIL PROTECTED] 
Sent: Tuesday, July 15, 2003 1:55 AM
Subject: [obm-l] Re: [obm-l] Re: [obm-l] 
Re:_[obm-l]_O_Erro_Comum_da_Generalização_(_Ou_induções_equivocadas_)_

Vc não se considera prepotente quando escreve coisas como "não admito que 
NINGUÉM, especialmente um ignorante em Matemática " e "exijo que o sr 
paulo recolha-se a sua insignificância "???
Sou uminútil completo,não sei de nada,nunca 
estudei,não estudonem sirvo pra nada,correto?
Certo,vc sabe de tudo,entende tudo que diz respeito à matemática,é o ser 
supremo que merece a atenção de todos e merece ser muito respeitado.

João Paulo


Re: [obm-l] Revista Eureka. No. 16

2003-07-15 Por tôpico matemandreca
Caro, Nelly e amigos da lista, 
Gostaria de saber como faço para que o colégio onde trabalho passe a receber 
a Eureka e maiores informações sobre as olimpiadas. Há alunos com interesse 
em participar. 
Desde já grato, André. 



Em 19 Jun 2003, [EMAIL PROTECTED] escreveu: 

Caros(as) amigos(as) da lista: 
 
Ja' estamos distribuindo as revistas Eureka No. 16 
para todas as Coordenacoes Regionais, 
Colegios Cadastrados na OBM e Assinantes. 
 
Abracos, Nelly. 
 
= 
Instruções para entrar na lista, sair da lista e usar a lista em 
http://www.mat.puc-rio.br/~nicolau/olimp/obm-l.html 
= 
 
-- 

_
Voce quer um iGMail protegido contra vírus e spams? 
Clique aqui: http://www.igmailseguro.ig.com.br
Ofertas imperdíveis! Link: http://www.americanas.com.br/ig/

=
Instruções para entrar na lista, sair da lista e usar a lista em
http://www.mat.puc-rio.br/~nicolau/olimp/obm-l.html
=


Re: [obm-l] Combinatoria (In off)

2003-07-15 Por tôpico Paulo Santa Rita
Ola Manuel e demais
colegas desta lista ... OBM-L,
1) A sua mensagem, nao obstante nao tratar de algum problema especifico, e 
muito boa. Ela realmente enriquece a Lista e e o tipo de OFF que espera-se 
que ocorra.

2) A Matematica continua e continuara sendo o tema desta Lista. Ainda que 
algumas pessoas mal educadas e mal formadas estejam - acredito firmemente - 
deliberadamente atentando contra estes objetivos originais, ela se mantera 
com a alta qualidade que nos, Professores, Pesquisadores e estudantes 
serios, almejamos e que lutamos para que tenha,

3) E verdade que o AXIOMA DO SUPREMO pode surgir como uma propriedade em 
alguma construcao particular dos numeros reais ou pode ser adotado como mais 
um axioma em outras construcoes. No livro de Analise I, Projeto Euclides. O 
Prof Elon adota este ultima postura, mas explicitamente cita outros obras 
nas quais os reais sao construidos e diz que uma tal construcao e um 
processo instrutivo. Em sintese, nao existe nenhuma razao mais forte para se 
adotar uma vertente ou outra, alem de crencas subjetivas e individuais.

4) A imensa maioria das teorias matematicas surgem de forma altamente 
intuitivas, pouco formais, somente rrecebendo um tratamento axiomatico 
posteriormente. Foi assim com o Calculo, com a Topologia, com a teoria dos 
grupos e com muitas ( talvez todas ! ) outras teorias. A intuicao vai na 
frente, descobre e orienta a pesquisa;  a formalizacao ou axiomatizacao vem 
depois e fundamenta com rigor as conquistas ja feitas. A primeira e a 
faculdade da descoberta, atributo do genio; a segunda, ferramente de prova e 
de resolucao de problemas, obra do talento.

Voce nao gostaria de apresentar aqui uma construcao dos reais, via cortes  
ou sequencias de Cauchy, por exemplo, e desta construcao derivar o TEOREMA 
DO SUPREMO ?

Um Abraco
Paulo Santa Rita
3,1154,150703

From: Manuel Valentim Pera [EMAIL PROTECTED]
Reply-To: [EMAIL PROTECTED]
To: [EMAIL PROTECTED]
Subject: Re: [obm-l] Combinatoria (In off)
Date: Mon, 14 Jul 2003 19:11:05 -0300 (EST)
Boa noite,

  Sobre o trecho:


 O segundo caso (mais geral) que você colocou, realmente merece uma
 demonstração, eu acho.
 Mas na minha cabeça, esse Princípio de Dirichlet seria uma coisa tão
 intuitiva que não precisaria de provas.
 Aí eu me embolo... Quando uma proposição precisa ser provada e quando se
 admite que ela é intuitiva o suficiente para ser aceita sem
demonstração?

  No sentido que as palavras tem em matematica (e matematica era, ate'
algum tempo atras, o assunto desta lista) sua duvida nao tem uma resposta
absoluta, exceto a trivial, DEPENDE do que foi admitido como axioma no
contexto de seu estudo, isto inclui, entre outras coisas mais mundanas,
quais os axiomas de teoria dos conjuntos que voce esta' admitindo. Tudo o
que nao for axioma precisa ser demonstrado.
  Eu nunca vi o principio de Dirichlet (ou pigeonhole) ser colocado como
axioma, entao precisa de uma demonstracao (se voce estiver admitindo os
postulados de Peanno para o conjunto dos naturais e ZF, isso sai
trivialmente, mas e' a demonstracao que e' trivial, nao a afirmacao. Alias
essa afirmacao tao trivial caracteriza, em muitos contextos, conjuntos
finitos), mas esta frase diz apenas isso: eu, na minha limitadissima
experiencia, nunca vi...
  Poucas afirmacoes sao tao evidentes (maldita palavra) como a do
Teorema da Curva de Jordan, se alguem conhecer alguma demonstracao
trivial dela, por favor, mostre-ma!
  Apenas um adendo, COM O AVISO DE IN-OFF EM MAIUSCULAS.

  Axioma, em matematica, nada tem a ver com intuitivo, ou evidente. os
axiomas das geometrias nao-euclideanas sao, do ponto de vista matematico
(outros pontos de vista deveriam ser assunto de bate papo em mesa de
botequim, coisas muito interessantes alias essas conversas, mas nao desta
lista), tao intuitivas quanto as euclideanas.
  Num exemplo concreto, falou-se nesta lista ha' nao muito tempo em
axioma do supremo para o conjunto dos Reais, isso ser um axioma so'
faz sentido numa teoria em que o conjunto dos numeros Reais (R) e'
apresentado axiomaticamente. Se voce quiser construir, por exemplo a
partir dos numeros racionais, esse conjunto isso deixa de ser axioma e
passa a ser uma PROPRIEDADE e precisa ser demonstrada.
  Hoje em dia pode parecer estranho falar-se em construcao de R, pois
o metodo axiomatico e' a unica forma que i conjunto dos Reais e'
apresentado (como dizia N. Rodrigues, toda unaminidade e' burra) e
construcoes de R sao temas desconhecidos dos dois primeiros anos de cursos
de graduacao em matematica, mas para pelo menos um grande Professor de
Matematica que eu conheco isso esta' longe de ser uma virtude do atual
modelo de ensino...
Desculpem o carater in-off do adendo, ele foge completamente dos objetivos
desta lista, mas em virtude de certas perolas recentes, que nem ao menos
vem com o aviso de in-off no subject penso que o professor Nicolau
perdoara' este deslize.
Manuel Garcia


Re: [obm-l] Problemas da IMO

2003-07-15 Por tôpico latino
Marcio,
achei legal essa sua solucao por complexos. Uma outra solucao
trivial (e acho que a de 99% dos participantes) seria a seguinte:

quad. APDR inscritivel  =  PR = AD.sen(BAC)
quad. CQRD inscritivel  =  RQ = DC.sen(ACB)

PR = RQ  =  AD/DC = sen(ACB)/sen(BAC) = AB/BC  (lei dos senos)   (*)

Sendo S e T os pontos de interseccao das bissetrizes internas dos
angulos ABC e ADC, respectivamente, com o lado AC, temos:

AS/SC  =  AB/BC  =  AD/DC  =  AT/TC   Logo, S = T
  (1)   (2)   (3)

(1) e (3) - teorema da bissetriz interna
(2) - por (*)

abracos,

#
# MSc. Edson Ricardo de A. Silva#
# Computer Graphics Group (CRAB)#
# Federal University of Ceara (UFC) #
#

On Tue, 15 Jul 2003, Marcio Afonso A. Cohen wrote:

 Eu sei que ninguem gosta muito disso, mas esse problema 4 (que eu ateh
 imagino que nao seja dificil por plana) eh bem simples na conta bruta.. Eh
 impressionante como complexos ajudam nos problemas de geometria da imo..
 aquele artigo da eureka 6 eh realmente muito util!

 Coloque o circuncentro na origem, e represente os vertices pelos
 complexos a,b,c,d, todos de modulo 1u.m.
 Reta ab: z+abz' = a+b
 Reta perpendicular a ab passando por d: z-abz'=d-abd'
 Logo, o ponto P eh 2p = [a+b+d-ab/d]
 Portanto, 2q = [a+c+d-ac/d] e 2r = [b+c+d-bc/d].
 Como p,q,r sao colineares (reta de simpson), e |p-q| = |q-r|:
 p-q = q-r, ou seja: b-c + ac/d - ab/d = a-b +bc/d-ac/d
 Arrumando: (b-c) - (a/d)(b-c) = (a-b) - (c/d)(a-b) sse (b-c)(d-a)=(a-b)(d-c)
 Tirando modulo, isso significa que BC*AD = AB*DC. E isso fecha o problema.
 De fato, sendo I o peh da bissetriz de ABC em AC, entao, AI/IC = AB/BC e vc
 quer provar que I eh peh da bissetriz de ADC, i.e, que AI/IC=AD/DC (teorema
 da bissetriz interna, ida e volta). Portanto, eh suficiente provar que AB*DC
 = AD*BC.

 Vou pensar nos outros agora, esse foi o que eu achei que seria mais
 facil.. (ja pensei no 2 e no 1 um pouco tmb..)


 - Original Message -
 From: [EMAIL PROTECTED]
 To: [EMAIL PROTECTED]
 Cc: [EMAIL PROTECTED]; [EMAIL PROTECTED]
 Sent: Monday, July 14, 2003 3:38 PM
 Subject: [obm-l] Problemas da IMO


 
 
  Prova da IMO retirada do Site http://www.mathlinks.go.ro/
 
  O Problema 1 é nois que mandou...
 
 
  First Day - 44th IMO 2003 Japan
 
  1. Let A be a 101-element subset of the set S={1,2,3,...,100}. Prove
 that
  there exist numbers t_1, t_2, ..., t_{100} in S such that the sets
 
  Aj = { x + tj | x is in A } for each j = 1, 2, ..., 100
 
  are pairwise disjoint.
 
 
  2. Find all pairs of positive integers (a,b) such that the number
 
  a^2 / ( 2ab^2-b^3+1) is also a positive integer.
 
  3. Given is a convex hexagon with the property that the segment connecting
 the
  middle points of each pair of opposite sides in the hexagon is  sqrt(3) /
 2
  times the sum of those sides' sum.
 
  Prove that the hexagon has all its angles equal to 120.
 
 
  Second Day - 44th IMO 2003 Japan
 
  4. Given is a cyclic quadrilateral ABCD and let P, Q, R be feet of the
  altitudes from D to AB, BC and CA respectively. Prove that if PR = RQ then
 the
  interior angle bisectors of the angles  ABC and  ADC are concurrent on
 AC.
 
  5. Let x1 = x2 = ... = xn be real numbers, n2.
 
  a) Prove the following inequality:
 
  (sum  ni,j=1 | xi - xj | ) 2 = 2/3 ( n^2 - 1 )sum ni,j=1 ( xi - xj)^2
 
  b) Prove that the equality in the inequality above is obtained if and only
 if
  the sequence (xk) is an arithemetical progression.
 
  6. Prove that for each given prime p there exists a prime q such that
 n^p - p
  is not divisible by q for each positive integer n.
 
 

 =
 Instruções para entrar na lista, sair da lista e usar a lista em
 http://www.mat.puc-rio.br/~nicolau/olimp/obm-l.html
 =

=
Instruções para entrar na lista, sair da lista e usar a lista em
http://www.mat.puc-rio.br/~nicolau/olimp/obm-l.html
=


Re: [obm-l] IMO - P1

2003-07-15 Por tôpico Marcio Afonso A. Cohen
Acho que consegui fazer  o 1o. Confiram ai e vejam se tem algum furo. O
2o eu realmente nao estou conseguindo.. Estou com alguma esperanca de fazer
o 5.. (o 3 eu tentei tmb, mas minhas contas estao muito grandes). Mandem
seus comentarios sobre a prova!
P1:
Note que (Ai inter Aj) != vazio sse existirem m,n tais que a_m + t_i =
a_n + t_j , i.e, a_m - a_n = t_j - t_i.
Vamos construir os t's indutivamente garantindo que isso nao acontece.
Existem binomial (101,2) = 5050 diferencas possiveis no conjunto A. Chame de
D={D1,D2,...D5050} o conjunto dessas diferencas (claro que algumas delas
podem ser iguais, mas temos |D| = 5050).
1. Escolha um t1 qualquer de S.
2. Agora quero garantir que t2-t1 e t1-t2 nao estao em D. Para isso, basta
escolher um elemento de S que nao esteja em
X1 = {t1+D1,t1+D2,...,t1+D5050}U{t1-D1, t1-D2,...,t1-D5050}. (pq se t2-t1
esta em D, entao t2=t1+Dk para algum k).
Isso eh facil pq |X1|=2.5050  |S|.

3. Agora vou escolher t3 em S garantindo que t3-t1, t1-t3, t3-t2, t2-t3 nao
estao em D.
Para isso, t3 nao pode estar em X1 e tmb nao pode estar em
X2 = {t2+D1,t2+D2,...,t2+D5050}U{t2-D1, t2-D2,...,t2-D5050}.
Isso eh facil, pq |X1 U X2| = 4.5050  |S|

Em geral, depois de escolhidos t1,t2,...,t_k-1, vou escolher t_k em S de
modo que ele nao esteja em nenhum dos conjuntos X1,X2,...,X_(k-1).
Para k=100, isso eh sempre possivel, pq |X1 U X2 U ... U X_(k-1)| =
2*(k-1)*5050 = 2*99*5050 = 00  10^6 = |S|.
(obs: X_s = {ts + D}U{ts-D}, na notacao usuao de x+A onde x eh um elemento e
A um conjunto).


Pronto. Foram escolhidos 100 t's tal que nao existe uma quadrupla (m,n,i,j)
tq a_m - a_n = t_j - t_i. (pois t_j - t_i esta sempre fora de D), e portanto
nunca se tem a_m + t_i = a_n + t_j, ou seja, as intersecoes sao todas vazias
de fato.

Abracos.



- Original Message -
From: [EMAIL PROTECTED]
To: [EMAIL PROTECTED]
Cc: [EMAIL PROTECTED]; [EMAIL PROTECTED]
Sent: Monday, July 14, 2003 3:38 PM
Subject: [obm-l] Problemas da IMO




 Prova da IMO retirada do Site http://www.mathlinks.go.ro/

 O Problema 1 é nois que mandou...


 First Day - 44th IMO 2003 Japan

 1. Let A be a 101-element subset of the set S={1,2,3,...,100}. Prove
that
 there exist numbers t_1, t_2, ..., t_{100} in S such that the sets

 Aj = { x + tj | x is in A } for each j = 1, 2, ..., 100

 are pairwise disjoint.


 2. Find all pairs of positive integers (a,b) such that the number

 a^2 / ( 2ab^2-b^3+1) is also a positive integer.

 3. Given is a convex hexagon with the property that the segment connecting
the
 middle points of each pair of opposite sides in the hexagon is  sqrt(3) /
2
 times the sum of those sides' sum.

 Prove that the hexagon has all its angles equal to 120.


 Second Day - 44th IMO 2003 Japan

 4. Given is a cyclic quadrilateral ABCD and let P, Q, R be feet of the
 altitudes from D to AB, BC and CA respectively. Prove that if PR = RQ then
the
 interior angle bisectors of the angles  ABC and  ADC are concurrent on
AC.

 5. Let x1 = x2 = ... = xn be real numbers, n2.

 a) Prove the following inequality:

 (sum  ni,j=1 | xi - xj | ) 2 = 2/3 ( n^2 - 1 )sum ni,j=1 ( xi - xj)^2

 b) Prove that the equality in the inequality above is obtained if and only
if
 the sequence (xk) is an arithemetical progression.

 6. Prove that for each given prime p there exists a prime q such that
n^p - p
 is not divisible by q for each positive integer n.



 -
 This mail sent through IMP: http://horde.org/imp/
 =
 Instruções para entrar na lista, sair da lista e usar a lista em
 http://www.mat.puc-rio.br/~nicolau/olimp/obm-l.html
 =


=
Instruções para entrar na lista, sair da lista e usar a lista em
http://www.mat.puc-rio.br/~nicolau/olimp/obm-l.html
=


[obm-l] Problemas IMO - Questao 4

2003-07-15 Por tôpico latino
So um pequeno detalhe... nao precisei usar o fato de ABCD ser incritivel
(pelo menos nao explicitamente). Alguem poderia comentar isso?

#
# MSc. Edson Ricardo de A. Silva#
# Computer Graphics Group (CRAB)#
# Federal University of Ceara (UFC) #
#

 achei legal essa sua solucao por complexos. Uma outra solucao
 trivial (e acho que a de 99% dos participantes) seria a seguinte:

 quad. APDR inscritivel  =  PR = AD.sen(BAC)
 quad. CQRD inscritivel  =  RQ = DC.sen(ACB)

 PR = RQ  =  AD/DC = sen(ACB)/sen(BAC) = AB/BC  (lei dos senos)   (*)

 Sendo S e T os pontos de interseccao das bissetrizes internas dos
 angulos ABC e ADC, respectivamente, com o lado AC, temos:

 AS/SC  =  AB/BC  =  AD/DC  =  AT/TC   Logo, S = T
   (1)   (2)   (3)

 (1) e (3) - teorema da bissetriz interna
 (2) - por (*)

 abracos,
=
Instruções para entrar na lista, sair da lista e usar a lista em
http://www.mat.puc-rio.br/~nicolau/olimp/obm-l.html
=


[obm-l] Re: [obm-l] RE: [obm-l] RE: [obm-l] Progressões: EXTREMAMENTE.......

2003-07-15 Por tôpico Henrique Patrício Sant'Anna Branco
 Eh claro que S(12 000) nao eh igual a 10 exatamente, Alexandre.

Segundo o Maple, S(12000) = 9,969919260.

Abraço,
Henrique.

=
Instruções para entrar na lista, sair da lista e usar a lista em
http://www.mat.puc-rio.br/~nicolau/olimp/obm-l.html
=


[obm-l] dificuldade

2003-07-15 Por tôpico Fabrício




Prove que:se a,b e c são lados de um triângulo e 
satisfaz a sentença a^2+b^2+c^2=9r^2, onde r é o 
raio da circunferência circunscrita, então esse triângulo é 
eqüilátero.


[obm-l] Progressões: EXTREMAMENTE.......

2003-07-15 Por tôpico MuriloRFL
segundo o maple,
S(15092689) = 100.000
S(1.509268*10^43)
  Eh claro que S(12 000) nao eh igual a 10 exatamente, Alexandre.

 Segundo o Maple, S(12000) = 9,969919260.

 Abraço,
 Henrique.

 =
 Instruções para entrar na lista, sair da lista e usar a lista em
 http://www.mat.puc-rio.br/~nicolau/olimp/obm-l.html
 =


=
Instruções para entrar na lista, sair da lista e usar a lista em
http://www.mat.puc-rio.br/~nicolau/olimp/obm-l.html
=


[obm-l] E-mail do Tengan sobre o IMO 6

2003-07-15 Por tôpico edmilson motta
Ei pessoal,

voces notaram que o problema 6 da prova e' uma
versao simplificada de um problema que eu e o Ed
mandamos em uma das listas de treinamento do ano
passado?  O problema da lista era algo assim:

Sejam a,r1 e p um primo.  Prove que existe um
primo q tal que (a mod q) tem ordem p^r.

Este e' o famoso lema de van der Waerden, que e'
utilizado na prova do teorema de reciprocidade
geral de Artin (mais detalhes, veja por exemplo
Lang, Algebraic Number Theory, pag. 200).

A minha solucao e' curta demais pra um problema 6 da
IMO,
entao gostaria de pedir que voces checassem a 
solucao.  Para nao irritar aqueles que ainda
nao pensaram no problema, vou deixar um espaco
em branco:















































mais em baixo...




































































mais um pouco...















































ta' chegando...
































Agora sim, vamos ao problema.  Em primeiro lugar,
olhando para uma raiz
primitiva 
de q, e' facil reduzir o problema a
provar que existe um primo q tal que
p mod q nao e' uma p-esima potencia, i.e.,
p^{(q-1)/p} mod q nao e' 1 mod q.

Considere

N = (p^p-1)/(p-1) = p^(p-1) + ... + p + 1

Se q e' um primo que divide N e p-1, entao de
N=p mod q, segue q=p, absurdo.  Entao para
todo primo q que divide N, p mod q tem ordem
exatamente p.  O problema acaba se p^2 nao
divide q-1, mas se todos os primos que dividem
N sao = 1 mod p^2, entao N = 1 mod p^2, o
que e' um absurdo.

Agora vejam: se no lema de van der Waerden
a=p, r=1, este e' exatamente o problema da IMO,
com algumas pequenas modificacoes!  A solucao
do problema da lista e' igualzinho `a
demonstracao acima.  Eu lembro que o Alex e o
Issao fizeram este problema, e acho que mais
alunos tambem acertaram.  Espero que os
pokemons tenham se lembrado do problema
durante a prova!

Estamos melhorando: um problema na IMO e uma
previsao acertada!  Alguem arrisca os proximos
numeros da loto?

Ate'
ET



__
Do you Yahoo!?
SBC Yahoo! DSL - Now only $29.95 per month!
http://sbc.yahoo.com
=
Instruções para entrar na lista, sair da lista e usar a lista em
http://www.mat.puc-rio.br/~nicolau/olimp/obm-l.html
=


[obm-l] Redução ao 1º quadrante

2003-07-15 Por tôpico Nelson alotiab
Olá a todos. Pra variar,gostaria de uma ajuda numa questãobem simples.Estareisinceramente grato por qualquerajuda.

[senx + cos(pi/2 - x)][cotg(x - pi) - cotg(2pi - x)]

Conseguiresponder essa questão:
cos(pi/2 - x) = senx
cotg(x - pi) = cotgx
cotg(2pi - x) = - cotgx
Substituindo:

[senx +senx)][cotgx - ( -cotgx)] = (2senx)(2cotgx) = (4senx)(cosx/senx) = 4cosx
Ok, mas, durante alguns dos meus "vôos", eu tinha pensado:
1º) cotg(x - pi), temos: x  pi;
2º) cotg(2pi - x), temos: x  2pi; 
3º) 1º) inter 2º) =pi  x  2pi 
e, agora, em cos(pi/2 - x),considerando3º), temos:
4º) cos(pi/2 - x) = cos[(pi/2 - x)  0]
Poderiamos também fazer outras inferências, mas, partindo dessa, eu só queria ratificar uma coisa:É Devido a simetria no ciclo trigonométrico que tornaa resolução "gabarito" válida, e faz omeu último raciocíniomais um "grande vôo"?

Obrigado pela atenção,
Nelson

Yahoo! Mail 
Mais espaço, mais segurança e gratuito: caixa postal de 6MB, antivírus, proteção contra spam.

[obm-l] Redução ao 1º quadrante

2003-07-15 Por tôpico Nelson alotiab

Olá a todos. Pra variar,gostaria de uma ajuda numa questãobem simples.Estareisinceramente grato por qualquerajuda.

[senx + cos(pi/2 - x)][cotg(x - pi) - cotg(2pi - x)]

Conseguiresponder essa questão:
cos(pi/2 - x) = senx
cotg(x - pi) = cotgx
cotg(2pi - x) = - cotgx
Substituindo:

[senx +senx)][cotgx - ( -cotgx)] = (2senx)(2cotgx) = (4senx)(cosx/senx) = 4cosx
Ok, mas, durante alguns dos meus "vôos", eu tinha pensado:
1º) cotg(x - pi), temos: x  pi;
2º) cotg(2pi - x), temos: x  2pi; 
3º) 1º) inter 2º) =pi  x  2pi 
e, agora, em cos(pi/2 - x),considerando3º), temos:
4º) cos(pi/2 - x) = cos[(pi/2 - x)  0]
Poderiamos também fazer outras inferências, mas, partindo dessa, eu só queria ratificar uma coisa:É Devido a simetria no ciclo trigonométrico que tornaa resolução "gabarito" válida, e faz omeu último raciocíniomais um "grande vôo"?

Obrigado pela atenção,
NelsonYahoo! Mail 
Mais espaço, mais segurança e gratuito: caixa postal de 6MB, antivírus, proteção contra spam.

[obm-l] Re: [obm-l] Seu crêisson!!! O_Erro_Comum...

2003-07-15 Por tôpico J.Paulo roxer ´til the end



Vcs acham que esse é um posicionamento sério de 
quem gosta de matemática e se preocupa com a educação?

Murilo,não entrei aqui pra ler coisas inúteis 
nem estou disposto a perder meu tempo lendo piadinhas 
ridículas.
Vá engrandecer seu intelecto e dar respostas 
mais sábias quanto às minhas indagações.

João Paulo

  - Original Message - 
  From: 
  MuriloRFL 
  
  To: OBM-Lista 
  Sent: Tuesday, July 15, 2003 10:26 
  AM
  Subject: [obm-l] Seu crêisson!!! 
  O_Erro_Comum...
  
  Seu sinhoJP,
  
  Nois, da 
  faucudadiseu Crêisson R$ ®,convidiamos vc pra nossia faucudadi 
  Nois temo aula especialica pra vc... Pra alunio reclamonico e griladiu!!! Temo 
  terapia de chókioA faucudade sónecessitia di sua 
  assinaturia Provanois num faiz pq nois num sabe mesmiu! nois 
  queremio éseu dinheirioE nem precisia escrivinha bestieiria!!! 
  Vc fiquia bem longi du lapisu.
  

  Gratilissimio,
  
  Faucudadi 
  seuCrêisson R$ ® 
  
  Agoria cum brilhiu nus olhius! 
  $-;-$
   
  X
  
  
  - Original Message - 
  From: J.Paulo 
  roxer ´til the end 
  To: [EMAIL PROTECTED] 
  Sent: Tuesday, July 15, 2003 1:55 AM
  Subject: [obm-l] Re: [obm-l] Re: [obm-l] 
  Re:_[obm-l]_O_Erro_Comum_da_Generalização_(_Ou_induções_equivocadas_)_
  
  Vc não se considera prepotente quando escreve coisas como "não admito que 
  NINGUÉM, especialmente um ignorante em Matemática " e "exijo que o sr 
  paulo recolha-se a sua insignificância "???
  Sou uminútil completo,não sei de nada,nunca 
  estudei,não estudonem sirvo pra nada,correto?
  Certo,vc sabe de tudo,entende tudo que diz respeito à matemática,é o ser 
  supremo que merece a atenção de todos e merece ser muito respeitado.
  
  João Paulo



Email.it, the professional e-mail, gratis per te: clicca qui 

Sponsor:
Iscriviti Gratis al primo corso on-line di Web Marketing in Italia!  Collegati subito a: http://www.internet-marketing.it/?EMAILk0403a
Clicca qui 

[obm-l] Análise Real

2003-07-15 Por tôpico bmat
Aqui vai um problema de Análise Real que está dando trabalho:

Prove que R - Q (o conjunto dos números Irracionais) não pode ser escrito
como uma união enumerável de conjuntos fechados.

Obrigado,
Bernardo



--
Use o melhor sistema de busca da Internet
Radar UOL - http://www.radaruol.com.br



=
Instruções para entrar na lista, sair da lista e usar a lista em
http://www.mat.puc-rio.br/~nicolau/olimp/obm-l.html
=


Re: [obm-l] Análise Real

2003-07-15 Por tôpico Manuel Valentim Pera
Bernardo,

  Boa tarde,

 
 Prove que R - Q (o conjunto dos números Irracionais) não pode ser escrito
 como uma união enumerável de conjuntos fechados.
 
 
  Se entendi o seu problema, ele pede para provar que, com a topologia
usual de R, nao existem subconjuntos fechados de R, F_1, F_2, ..., F_n,
..., tais que a reuniao dos F_n seja R-Q, certo?

  Se for isso, suponha por abusrdo, que isso e' falso e seja, para cada n,
O_n = R - F_n.

  O_n e' aberto, qualquer que seja n, e e' facil ver que a interseccao dos
O_n e' Q.

  Como Q e' denso em R, e' claro que cada O_n e' denso em R.

  Entao, como R e' um espaco de Baire (por ser completo), segue-se do
teorema de Baire que a interseccao dos O_n e' um espaco de Baire. 

  Mas isto e' um absurdo, pois Q e' enumeravel, e portanto e' a reuniao
enumeravel de fechados sem interior nao podendo assim ser de Baire.

Manuel Garcia


=
Instruções para entrar na lista, sair da lista e usar a lista em
http://www.mat.puc-rio.br/~nicolau/olimp/obm-l.html
=


[obm-l] Referencia Bibliografica (era: Combinatoria (In off))

2003-07-15 Por tôpico Manuel Valentim Pera
Paulo,

  Boa tarde,

On Tue, 15 Jul 2003, Paulo Santa Rita wrote:

 
 Voce nao gostaria de apresentar aqui uma construcao dos reais, via cortes  
 ou sequencias de Cauchy, por exemplo, e desta construcao derivar o TEOREMA 
 DO SUPREMO ?
 

 Nao teria sentido fazer isso aqui, mas recomendo a leitura dos excelentes
textos abaixo em que isso esta' bem feito, muito mais bem feito do que
qualquer coisa que eu pudesse esbocar aqui (eles podem ser encontrados em
bibliotecas de faculdades de Matematica):

  (A) Para construcoes via cortes de Dedekind: 

  Rudin, W.- Priciples of Real Analysis-3rd edition, McGraw Hill
International Editions (da segunda edicao deste livro ha', infelizmente,
uma traducao, mas essa edicao nao tras a cosntrucao de R). Veja o apendice
ao capitulo 1.

  Spivak, M.- Calculus (vol II) - Editorial Reverte, veja o apendice 1 
deste texto.

  (B) Para construcoes atraves de sequencias de Cauchy (particularmente
gosto mais deste enfoque):

  Esta' feita na seccao 5 do livro

  Hewitt, E.  Stromberg, K.- Real and Abstract Analysis - Springer.
  
Um ultimo comentario, historico, a construcao por cortes e' devida a
Dedekind (foi publicada pela promeira vez em 1872) e por sequencias de
Cauchy e' devida a Cantor, sendo originariamente publicada tambem em 1872.

Manuel Garcia

=
Instruções para entrar na lista, sair da lista e usar a lista em
http://www.mat.puc-rio.br/~nicolau/olimp/obm-l.html
=


[obm-l] Re: [obm-l] Análise Real

2003-07-15 Por tôpico bmat
Manuel,
Boa tarde.

Muito boa a solução para este problema, mas eu não conheço o teorema de
Baire, nem lembro muito bem o que era um espaço de Baire. Mas o pior é que
este problema tinha um corolário: conclua que Q não é a reunião enumerável
de abertos... então eu suponho que deve haver outro meio para resolver este
problema. Para ser mais completo, deixo agora a referência:
Curso de Análise, vol 1 - Elon Lages Lima
Capítulo 5 (Topologia da Reta) - exercício 55

Muito obrigado pela atenção,
Bernardo

-- Mensagem original --

Bernardo,

  Boa tarde,

 
 Prove que R - Q (o conjunto dos números Irracionais) não pode ser escrito
 como uma união enumerável de conjuntos fechados.
 
 
  Se entendi o seu problema, ele pede para provar que, com a topologia
usual de R, nao existem subconjuntos fechados de R, F_1, F_2, ..., F_n,
..., tais que a reuniao dos F_n seja R-Q, certo?

  Se for isso, suponha por abusrdo, que isso e' falso e seja, para cada
n,
O_n = R - F_n.

  O_n e' aberto, qualquer que seja n, e e' facil ver que a interseccao
dos
O_n e' Q.

  Como Q e' denso em R, e' claro que cada O_n e' denso em R.

  Entao, como R e' um espaco de Baire (por ser completo), segue-se do
teorema de Baire que a interseccao dos O_n e' um espaco de Baire. 

  Mas isto e' um absurdo, pois Q e' enumeravel, e portanto e' a reuniao
enumeravel de fechados sem interior nao podendo assim ser de Baire.

Manuel Garcia


=
Instruções para entrar na lista, sair da lista e usar a lista em
http://www.mat.puc-rio.br/~nicolau/olimp/obm-l.html
=




--
Use o melhor sistema de busca da Internet
Radar UOL - http://www.radaruol.com.br



=
Instruções para entrar na lista, sair da lista e usar a lista em
http://www.mat.puc-rio.br/~nicolau/olimp/obm-l.html
=


[obm-l] curvas

2003-07-15 Por tôpico Marcelo Souza
Gente,
alguém pode me ajudar a resolver os problemas
1. Sejam e uma elipse e h uma hiperbole tendo focos em comum. Mostre que e e 
h se cortam perpedicularmente.
(alguém podria exibir uma solução usando derivadas, é pq estou tentando e 
não consegui)

2.Seja c:I-R^2 uma curva com segunda derivada e tq c'(t) dif de  (0,0) para 
todo t. Suponha ainda que a aceleração normal não se anule. A evoluta de c é 
a curva
a:I-R^2
 t -- (x(t),y(t))+(1/k(t)(x'(t)^2+y'(t)^2)^(1/2))*(-y'(t),x'(t)),
onde c(t)=(x(t),y(t)) e k(t) = (-y'x''+x'y'')/(x'^2+y'^2)^(3/2) (omiti t)

I) Mostre que a reta tangente a a em a(t) coincide com a normal a c em c(t).

valeu
[]'s, Marcelo
_
MSN Messenger: instale grátis e converse com seus amigos. 
http://messenger.msn.com.br

=
Instruções para entrar na lista, sair da lista e usar a lista em
http://www.mat.puc-rio.br/~nicolau/olimp/obm-l.html
=


[obm-l] Re: [obm-l] Análise Real

2003-07-15 Por tôpico Manuel Valentim Pera
Bernardo,

  Boa tarde,

  Só dois comentários:

  (1) Há algo estranho com o corolário, ele é completamente trivial,
mas não sei como concluir do exercício original esse resultado. Veja o
seguinte, Q não pode ser a renuião enumerável de abertos, simplesmente
porque cada aberto não vazio de R contém um inervalo aberto (a,b) não
vazio. Logo se Q fosse uma reunião de abertos (enumerável ou não) Q
conteria (a,b). Iso é absurdo pois R-Q é denso em R. Talvez o corolário
seja Q não é a intercecção enumerável de abertos. De fato isso segue-se
imediatamente do exercício proposto, por passagem ao complementar.

  (2) Não sei exatamente o contexto em que o exercício apareceu, às vezes
quando se fala em R, esconde-se quando se está usando Baire. Você precisa
saber alguma coisa, por exemplo que R não pode ser escrito como reunião
enumerável de fechados sem interior [pode chamar isso propriedade de Baire
da reta] ou algo equivalente para fazer o exercício (o que foi usado na
demonstração do outro email foi algo equivalente). Se você souber dessa
propriedade que enunciei acima, uma demonstração alternativa (que, no
fundo é exatamente igual) é a seguinte.

Suponha, por absurdo, que existem subconjuntos fechados de R, F_1,
F_2,..., F_n,... tais que a reunião de todos os F_n seja R-Q.

Como Q não tem interior (pois nenhum intervalo aberto da reta, não
vazio, está contido em Q) segue-se que cada F_n tem interior vazio.

   Qomo Q é enumerável tome {q_k, k em N} uma enumeração de Q e defina
T_j={q_j}, j=1,2,...

  Claro que cada T_j é fechado e de interior vazio.

  Então R = (R-Q) U Q seria a reunião dos F_n com os T_j. Então ter-se-ia
escrito R como uma reunião enumerável de fechados sem interior, o que
contraria a aupramencionada propriedade de Baire da reta.

Manuel Garcia


 On Tue, 15 Jul 2003 [EMAIL PROTECTED] wrote:

 Manuel,
 Boa tarde.

 Muito boa a solução para este problema, mas eu não conheço o teorema de
 Baire, nem lembro muito bem o que era um espaço de Baire. Mas o pior é que
 este problema tinha um corolário: conclua que Q não é a reunião enumerável
 de abertos... então eu suponho que deve haver outro meio para resolver este
 problema. Para ser mais completo, deixo agora a referência:
 Curso de Análise, vol 1 - Elon Lages Lima
 Capítulo 5 (Topologia da Reta) - exercício 55

 Muito obrigado pela atenção,
 Bernardo


=
Instruções para entrar na lista, sair da lista e usar a lista em
http://www.mat.puc-rio.br/~nicolau/olimp/obm-l.html
=


[obm-l] Seu crêisson!!! O_Erro_Comum...

2003-07-15 Por tôpico MuriloRFL



Joao Paulo,

Se vc nao entrou aki para ler coisas inuteis, 
eu tmb nao
Enem gastovocabulariopara 
responder tuas indagações (Talvez o Vocabulo Seu 
Crêisson!)...
Se vc sentiu-se ridicularizado é por que a 
carapulsa serviu...
Vai estudar... melhor do que vc 
ficar perdendo tempo respondendo piadinhas 
ridiculas...
O caracter ironico deste meu ridiculo ainda vai 
percurtir no seu ego.
Pq a verdade nao morree nem nasce, ela não 
se cria, ela existe...
Quem disse que eu me preoucupo com a educação ou 
tenho um posicionamento sério para com os auto-excluidos e 
auto-incapacitados

Pra vc, vale a pena repetir:

"Quem 
conheceooutro é inteligente,
Quem conhece a si mesmo 
é iluminado!
Quem vence o outro é 
forte,
Quem vence a si mesmo é 
invencível"

Mao-tse-tung

Sem mais, 
Indignado,

Murilo Lima

P.s - Pq seu e-mail é o unico cor de lilas 
gay?!
P.s2 - "Quem sabe faz a hora, nao espera 
acontecer"
P.s3 - Pensava eu na minha ignorância, que esta 
lista fosse direcionada à OBM, não à "Como lecionar Matematica e outros bixos no 
ensino médio"
P.s4 - Me desculpem a todos da lista pelo tempo 
perdido neste e-mail...


- Original Message - 
From: J.Paulo roxer 
´til the end 
To: [EMAIL PROTECTED] 
Sent: Tuesday, July 15, 2003 4:13 PM
Subject: [obm-l] Re: [obm-l] Seu crêisson!!! 
O_Erro_Comum...

Vcs acham que esse é um posicionamento sério de 
quem gosta de matemática e se preocupa com a educação?

Murilo,não entrei aqui pra ler coisas inúteis 
nem estou disposto a perder meu tempo lendo piadinhas 
ridículas.
Vá engrandecer seu intelecto e dar respostas 
mais sábias quanto às minhas indagações.

João Paulo

  - Original Message - 
  From: 
  MuriloRFL 
  
  To: OBM-Lista 
  Sent: Tuesday, July 15, 2003 10:26 
  AM
  Subject: [obm-l] Seu crêisson!!! 
  O_Erro_Comum...
  
  Seu sinhoJP,
  
  Nois, da 
  faucudadiseu Crêisson R$ ®,convidiamos vc pra nossia faucudadi 
  Nois temo aula especialica pra vc... Pra alunio reclamonico e griladiu!!! Temo 
  terapia de chókioA faucudade sónecessitia di sua 
  assinaturia Provanois num faiz pq nois num sabe mesmiu! nois 
  queremio éseu dinheirioE nem precisia escrivinha bestieiria!!! 
  Vc fiquia bem longi du lapisu.
  

  Gratilissimio,
  
  Faucudadi 
  seuCrêisson R$ ® 
  
  Agoria cum brilhiu nus olhius! 
  $-;-$
   
  X
  
  
  - Original Message - 
  From: J.Paulo 
  roxer ´til the end 
  To: [EMAIL PROTECTED] 
  Sent: Tuesday, July 15, 2003 1:55 AM
  Subject: [obm-l] Re: [obm-l] Re: [obm-l] 
  Re:_[obm-l]_O_Erro_Comum_da_Generalização_(_Ou_induções_equivocadas_)_
  
  Vc não se considera prepotente quando escreve coisas como "não admito que 
  NINGUÉM, especialmente um ignorante em Matemática " e "exijo que o sr 
  paulo recolha-se a sua insignificância "???
  Sou uminútil completo,não sei de nada,nunca 
  estudei,não estudonem sirvo pra nada,correto?
  Certo,vc sabe de tudo,entende tudo que diz respeito à matemática,é o ser 
  supremo que merece a atenção de todos e merece ser muito respeitado.
  
  João Paulo
Email.it, the professional e-mail, 
gratis per te: clicca qui Sponsor:Iscriviti Gratis al primo corso 
on-line di Web Marketing in Italia! Collegati subito a: 
http://www.internet-marketing.it/?EMAILk0403aClicca qui 


[obm-l] polinomios

2003-07-15 Por tôpico Eduardo Henrique Leitner
Fundamentos de Matemática Elementar, volume 6

Gelson Iezzi

145. Seja P(x) um polinômio de 5^o grau que satisfaz as condições:

1 = P(1) = P(2) = P(3) = P(4) = P(5)
e
0 = P(6).

Qual o valor de P(0)?



eu tentei fazer pelo sistema... mas putz... sem condições...
=
Instruções para entrar na lista, sair da lista e usar a lista em
http://www.mat.puc-rio.br/~nicolau/olimp/obm-l.html
=


Re: [obm-l] dificuldade

2003-07-15 Por tôpico Marcelo Rufino de Oliveira





  - Original Message - 
  From: 
  Fabrício 
  
  To: [EMAIL PROTECTED] 
  Sent: Tuesday, July 15, 2003 2:31 
PM
  Subject: [obm-l] dificuldade
  
  
  Prove que:se a,b e c são lados de um triângulo e 
  satisfaz a sentença a^2+b^2+c^2=9r^2, onde r é 
  o raio da circunferência circunscrita, então esse triângulo é 
  eqüilátero.
  
  
  1a. Solução (esperta):
  Usando o fato de que a distância entre circuncentro 
  O e o baricentro G de um triângulo é igual a R^2 - (a^2 + b^2 + c^2)/9 temos 
  que O = G e segue diretamente que o triângulo é 
  equilátero.
  
  
  2a. Solução (lusitana):
  Pela Lei dos Senos a/R = 2.sen A, b/R = 
  2.sen B e c/R = 2.sen C.
  Assim: 9 = 4.sen^2 A + 4.sen^2 B + 4.sen^2 C = 
  2 - 2.cos 2A + 2 - 2.cos 2B + 2 - cos 2C 
  =
  cos 2A + cos 2B + cos 2C = - 
  3/2agora é braço, use trigonometria para provar 
  que a única solução desta equaçãoé A = B = C.
  
  Falou,
  Marcelo Rufino de Oliveira
  
  


[obm-l] Transformada de Laplace

2003-07-15 Por tôpico Marcus Alexandre Nunes



Como fazer para calcular as trnasformadas de 
Laplace das funções

cos(mt)sen(nt)

e

exp(-2x)(cosx)^2 ?

Marcus Alexandre Nunes[EMAIL PROTECTED]http://darwingauss.blogspot.comUIN 
114153703


Re: [obm-l] polinomios

2003-07-15 Por tôpico Marcelo Rufino de Oliveira
Seja Q(x) = P(x) - 1   =   Q(1) = Q(2) = Q(3) = Q(4) = Q(5) = 0. Como Q(x)
também possui grau cinco, 1, 2, 3, 4, e 5 são as cinco raízes de Q(x)   =
Q(x) = A(x - 1)(x - 2)(x - 3)(x - 4)(x - 5) (1)

P(6) = 0   =   Q(6) = - 1

Aplicando x = 6 em (1)   =   - 1 = A.5.4.3.2.1   =   A = - 1/120   =
Q(x) = - (x - 1)(x - 2)(x - 3)(x - 4)(x - 5)/120   =
P(x) = 1 - (x - 1)(x - 2)(x - 3)(x - 4)(x - 5)/120

Assim,  P(0) = 1 - (-1)(-2)(-3)(-4)(-5)/120   =   P(0) = 2



Marcelo Rufino de Oliveira


- Original Message -
From: Eduardo Henrique Leitner [EMAIL PROTECTED]
To: lista de matemática [EMAIL PROTECTED]
Sent: Tuesday, July 15, 2003 6:52 PM
Subject: [obm-l] polinomios


 Fundamentos de Matemática Elementar, volume 6

 Gelson Iezzi

 145. Seja P(x) um polinômio de 5^o grau que satisfaz as condições:

 1 = P(1) = P(2) = P(3) = P(4) = P(5)
 e
 0 = P(6).

 Qual o valor de P(0)?



 eu tentei fazer pelo sistema... mas putz... sem condições...
 =
 Instruções para entrar na lista, sair da lista e usar a lista em
 http://www.mat.puc-rio.br/~nicolau/olimp/obm-l.html
 =

=
Instruções para entrar na lista, sair da lista e usar a lista em
http://www.mat.puc-rio.br/~nicolau/olimp/obm-l.html
=


[obm-l] Transformadas de Laplace II

2003-07-15 Por tôpico Marcus Alexandre Nunes



Eu esqueci de dizer no outro e-mail: se for 
possível colocar, de modo genérico, a regra ou propriedade que foi usada pra 
resolver o problema, eu agradeço muito.

Marcus Alexandre Nunes[EMAIL PROTECTED]http://darwingauss.blogspot.comUIN 
114153703


[obm-l] Desafio Dificio (raposas e galinhas)

2003-07-15 Por tôpico MuriloRFL



"Vc é capaz de distribuir em um tabuleiro 5x5, 5 
raposas e 3 galinhas de tal forma que nenhuma raposa ataque alguma das 3 
galinhas? Sabe-se queas raposasse movimentamcomo rainhas no 
jogo de xadrez."


[obm-l] Problemas IMO - Questao 4

2003-07-15 Por tôpico Rodrigo Villard Milet
Parece estar certo... Eu fiz uma solução legalzinha... segue :



Deixo um espaço em branco...





.








.

















Veja que podemos supor que P está fora de AB e Q está dentro de BC, pois
como A+C= 180, um dos A ou C deve ser agudo e o outro obtuso. [XYZ] = área
do triângulo XYZ.
Veja que [ACQ]=[APC] (pois como PR=QR, então [APR]=[ARQ] e [CPR]=[CRQ], já
que P,Q e R são colineares - reta de Simson)
Logo [BCP]-[ABC]=[ABC]-[ABQ], então BP.BC-BA.BC=BA.BC-BA.BQ, o que é
equivalente a BA.(BC-BQ) = BC.(BP-BA) ... BA.CQ=BC.PA. Como PA/CQ =
AD.cosC/CD.cosC  = AD/CD, segue que BA/BC=AD/CD, o que finaliza o problema.

Abraços,
 Villard
-Mensagem original-
De: [EMAIL PROTECTED] [EMAIL PROTECTED]
Para: [EMAIL PROTECTED] [EMAIL PROTECTED]
Data: Terça-feira, 15 de Julho de 2003 13:00
Assunto: [obm-l] Problemas IMO - Questao 4


So um pequeno detalhe... nao precisei usar o fato de ABCD ser incritivel
(pelo menos nao explicitamente). Alguem poderia comentar isso?

#
# MSc. Edson Ricardo de A. Silva#
# Computer Graphics Group (CRAB)#
# Federal University of Ceara (UFC) #
#

 achei legal essa sua solucao por complexos. Uma outra solucao
 trivial (e acho que a de 99% dos participantes) seria a seguinte:

 quad. APDR inscritivel  =  PR = AD.sen(BAC)
 quad. CQRD inscritivel  =  RQ = DC.sen(ACB)

 PR = RQ  =  AD/DC = sen(ACB)/sen(BAC) = AB/BC  (lei dos senos)   (*)

 Sendo S e T os pontos de interseccao das bissetrizes internas dos
 angulos ABC e ADC, respectivamente, com o lado AC, temos:

 AS/SC  =  AB/BC  =  AD/DC  =  AT/TC   Logo, S = T
   (1)   (2)   (3)

 (1) e (3) - teorema da bissetriz interna
 (2) - por (*)

 abracos,
=
Instruções para entrar na lista, sair da lista e usar a lista em
http://www.mat.puc-rio.br/~nicolau/olimp/obm-l.html
=


=
Instruções para entrar na lista, sair da lista e usar a lista em
http://www.mat.puc-rio.br/~nicolau/olimp/obm-l.html
=


RE: [obm-l] Transformada de Laplace

2003-07-15 Por tôpico Leandro Lacorte Recôva









Porque voce nao usa as
formulas : 



Sin(a + b) =
sin(a).cos(b) + cos(a)sin(b)

Sin(a  b) =
sin(a).cos(b)  cos(b)sin(a) 



Some (1) e (2)



Sin(a+b) + sin(a-b) =
2.sin(a).cos(b)  



Portanto, g(t) =  cos(mt).sin(nt)
= (sin(m+n)t + sin(m-n)t)/2 



Assim, sabendo que a
transformada de sin(at) = (a)/(s^2+a^2) entao,



G(s) = ((m+n)/(s^2+(m+n)^2)
+ (m-n)/(s^2+(m-n)^2))*1/2   (Lembre que a transformada e linear e tirei a
transformada de cada membro de g(t).



Se errei algum sinal,
pode corrigir, mas o raciocinio e esse. 



Leandro

Los Angeles, CA



-Original Message-
From:
[EMAIL PROTECTED] [mailto:[EMAIL PROTECTED] On Behalf Of Marcus Alexandre Nunes
Sent: Tuesday, July
 15, 2003 3:45 PM
To: Lista OBM
Subject: [obm-l] Transformada de
Laplace





Como fazer para calcular as trnasformadas de Laplace das
funções











cos(mt)sen(nt)











e











exp(-2x)(cosx)^2 ?












Marcus Alexandre Nunes
[EMAIL PROTECTED]
http://darwingauss.blogspot.com
UIN 114153703










Re: [obm-l] polinomios

2003-07-15 Por tôpico Fernando Henrique Ferraz Pereira da Rosa
 Incidentalmente há uns 2 anos também 'trombei' com esse problema e tive a
paciência de resolver o sistema para achar a resposta. De qualquer forma
postei uma mensagem para a lista e me mandaram uma sugestão mais simples:
 http://www.mat.puc-rio.br/~nicolau/olimp/obm-l.200107/msg4.html


-- 
[]'s

+++ GMX - Mail, Messaging  more  http://www.gmx.net +++

Jetzt ein- oder umsteigen und USB-Speicheruhr als Prämie sichern!

=
Instruções para entrar na lista, sair da lista e usar a lista em
http://www.mat.puc-rio.br/~nicolau/olimp/obm-l.html
=


[obm-l] Matrizes

2003-07-15 Por tôpico ghaeser
Prove que não existem matrizes reais A e B tal que AB-BA=I

Mathematicus nascitur, non fit
Matemáticos não são feitos, eles nascem
---
Gabriel Haeser
www.gabas.cjb.net


--
Use o melhor sistema de busca da Internet
Radar UOL - http://www.radaruol.com.br



=
Instruções para entrar na lista, sair da lista e usar a lista em
http://www.mat.puc-rio.br/~nicolau/olimp/obm-l.html
=


Re: [obm-l] Problemas da IMO

2003-07-15 Por tôpico Marcio Afonso A. Cohen
Realmente, sua solucao me parece perfeita.. Alem de nao usar que o
quadrilatero eh inscritivel.. legal.
Voce pensou nos outros? Pensei bem no 2 e no 3, mas nao consegui fechar
nenhum.. O 3 eu acredito que seja alguma desigualdade virando igualdade, e
quero tentar mais pra ver se da certo..

- Original Message -
From: [EMAIL PROTECTED]
To: [EMAIL PROTECTED]
Sent: Tuesday, July 15, 2003 12:05 PM
Subject: Re: [obm-l] Problemas da IMO


 Marcio,
 achei legal essa sua solucao por complexos. Uma outra solucao
 trivial (e acho que a de 99% dos participantes) seria a seguinte:

 quad. APDR inscritivel  =  PR = AD.sen(BAC)
 quad. CQRD inscritivel  =  RQ = DC.sen(ACB)

 PR = RQ  =  AD/DC = sen(ACB)/sen(BAC) = AB/BC  (lei dos senos)   (*)

 Sendo S e T os pontos de interseccao das bissetrizes internas dos
 angulos ABC e ADC, respectivamente, com o lado AC, temos:

 AS/SC  =  AB/BC  =  AD/DC  =  AT/TC   Logo, S = T
   (1)   (2)   (3)

 (1) e (3) - teorema da bissetriz interna
 (2) - por (*)

 abracos,

 #
 # MSc. Edson Ricardo de A. Silva#
 # Computer Graphics Group (CRAB)#
 # Federal University of Ceara (UFC) #
 #

 On Tue, 15 Jul 2003, Marcio Afonso A. Cohen wrote:

  Eu sei que ninguem gosta muito disso, mas esse problema 4 (que eu
ateh
  imagino que nao seja dificil por plana) eh bem simples na conta bruta..
Eh
  impressionante como complexos ajudam nos problemas de geometria da imo..
  aquele artigo da eureka 6 eh realmente muito util!
 
  Coloque o circuncentro na origem, e represente os vertices pelos
  complexos a,b,c,d, todos de modulo 1u.m.
  Reta ab: z+abz' = a+b
  Reta perpendicular a ab passando por d: z-abz'=d-abd'
  Logo, o ponto P eh 2p = [a+b+d-ab/d]
  Portanto, 2q = [a+c+d-ac/d] e 2r = [b+c+d-bc/d].
  Como p,q,r sao colineares (reta de simpson), e |p-q| = |q-r|:
  p-q = q-r, ou seja: b-c + ac/d - ab/d = a-b +bc/d-ac/d
  Arrumando: (b-c) - (a/d)(b-c) = (a-b) - (c/d)(a-b) sse
(b-c)(d-a)=(a-b)(d-c)
  Tirando modulo, isso significa que BC*AD = AB*DC. E isso fecha o
problema.
  De fato, sendo I o peh da bissetriz de ABC em AC, entao, AI/IC = AB/BC e
vc
  quer provar que I eh peh da bissetriz de ADC, i.e, que AI/IC=AD/DC
(teorema
  da bissetriz interna, ida e volta). Portanto, eh suficiente provar que
AB*DC
  = AD*BC.
 
  Vou pensar nos outros agora, esse foi o que eu achei que seria mais
  facil.. (ja pensei no 2 e no 1 um pouco tmb..)
 
 
  - Original Message -
  From: [EMAIL PROTECTED]
  To: [EMAIL PROTECTED]
  Cc: [EMAIL PROTECTED]; [EMAIL PROTECTED]
  Sent: Monday, July 14, 2003 3:38 PM
  Subject: [obm-l] Problemas da IMO
 
 
  
  
   Prova da IMO retirada do Site http://www.mathlinks.go.ro/
  
   O Problema 1 é nois que mandou...
  
  
   First Day - 44th IMO 2003 Japan
  
   1. Let A be a 101-element subset of the set S={1,2,3,...,100}.
Prove
  that
   there exist numbers t_1, t_2, ..., t_{100} in S such that the sets
  
   Aj = { x + tj | x is in A } for each j = 1, 2, ..., 100
  
   are pairwise disjoint.
  
  
   2. Find all pairs of positive integers (a,b) such that the number
  
   a^2 / ( 2ab^2-b^3+1) is also a positive integer.
  
   3. Given is a convex hexagon with the property that the segment
connecting
  the
   middle points of each pair of opposite sides in the hexagon is
sqrt(3) /
  2
   times the sum of those sides' sum.
  
   Prove that the hexagon has all its angles equal to 120.
  
  
   Second Day - 44th IMO 2003 Japan
  
   4. Given is a cyclic quadrilateral ABCD and let P, Q, R be feet of the
   altitudes from D to AB, BC and CA respectively. Prove that if PR = RQ
then
  the
   interior angle bisectors of the angles  ABC and  ADC are concurrent
on
  AC.
  
   5. Let x1 = x2 = ... = xn be real numbers, n2.
  
   a) Prove the following inequality:
  
   (sum  ni,j=1 | xi - xj | ) 2 = 2/3 ( n^2 - 1 )sum ni,j=1 ( xi - xj)^2
  
   b) Prove that the equality in the inequality above is obtained if and
only
  if
   the sequence (xk) is an arithemetical progression.
  
   6. Prove that for each given prime p there exists a prime q such that
  n^p - p
   is not divisible by q for each positive integer n.
  
  
 
 
=
  Instruções para entrar na lista, sair da lista e usar a lista em
  http://www.mat.puc-rio.br/~nicolau/olimp/obm-l.html
 
=
 
 =
 Instruções para entrar na lista, sair da lista e usar a lista em
 http://www.mat.puc-rio.br/~nicolau/olimp/obm-l.html
 =


=
Instruções para entrar na lista, sair da lista e usar a lista em
http://www.mat.puc-rio.br/~nicolau/olimp/obm-l.html

Re: [obm-l] Matrizes

2003-07-15 Por tôpico A. C. Morgado
Traço AB = traço BA
traço (AB-BA)=0
traço I = n
[EMAIL PROTECTED] wrote:

Prove que não existem matrizes reais A e B tal que AB-BA=I

Mathematicus nascitur, non fit
Matemáticos não são feitos, eles nascem
---
Gabriel Haeser
www.gabas.cjb.net
--
Use o melhor sistema de busca da Internet
Radar UOL - http://www.radaruol.com.br


=
Instruções para entrar na lista, sair da lista e usar a lista em
http://www.mat.puc-rio.br/~nicolau/olimp/obm-l.html
=
 

=
Instruções para entrar na lista, sair da lista e usar a lista em
http://www.mat.puc-rio.br/~nicolau/olimp/obm-l.html
=


Re: [obm-l] polinomios

2003-07-15 Por tôpico Eduardo Henrique Leitner
valeu cara! eu realmente nao hia pensar nisso...

On Tue, Jul 15, 2003 at 07:46:29AM -0300, Marcelo Rufino de Oliveira wrote:
 Seja Q(x) = P(x) - 1   =   Q(1) = Q(2) = Q(3) = Q(4) = Q(5) = 0. Como Q(x)
 também possui grau cinco, 1, 2, 3, 4, e 5 são as cinco raízes de Q(x)   =
 Q(x) = A(x - 1)(x - 2)(x - 3)(x - 4)(x - 5) (1)
 
 P(6) = 0   =   Q(6) = - 1
 
 Aplicando x = 6 em (1)   =   - 1 = A.5.4.3.2.1   =   A = - 1/120   =
 Q(x) = - (x - 1)(x - 2)(x - 3)(x - 4)(x - 5)/120   =
 P(x) = 1 - (x - 1)(x - 2)(x - 3)(x - 4)(x - 5)/120
 
 Assim,  P(0) = 1 - (-1)(-2)(-3)(-4)(-5)/120   =   P(0) = 2
 
 
 
 Marcelo Rufino de Oliveira
 
 
 - Original Message -
 From: Eduardo Henrique Leitner [EMAIL PROTECTED]
 To: lista de matemática [EMAIL PROTECTED]
 Sent: Tuesday, July 15, 2003 6:52 PM
 Subject: [obm-l] polinomios
 
 
  Fundamentos de Matemática Elementar, volume 6
 
  Gelson Iezzi
 
  145. Seja P(x) um polinômio de 5^o grau que satisfaz as condições:
 
  1 = P(1) = P(2) = P(3) = P(4) = P(5)
  e
  0 = P(6).
 
  Qual o valor de P(0)?
 
 
 
  eu tentei fazer pelo sistema... mas putz... sem condições...
  =
  Instruções para entrar na lista, sair da lista e usar a lista em
  http://www.mat.puc-rio.br/~nicolau/olimp/obm-l.html
  =
 
 =
 Instruções para entrar na lista, sair da lista e usar a lista em
 http://www.mat.puc-rio.br/~nicolau/olimp/obm-l.html
 =
=
Instruções para entrar na lista, sair da lista e usar a lista em
http://www.mat.puc-rio.br/~nicolau/olimp/obm-l.html
=


Re: [obm-l] Transformada de Laplace

2003-07-15 Por tôpico Marcus Alexandre Nunes



Bah, nem tinha me ligado em usar estas 
propriedades trigonométricas. Valeu Leandro.

Marcus Alexandre Nunes[EMAIL PROTECTED]http://darwingauss.blogspot.comUIN 
114153703


Re: [obm-l] polinomios

2003-07-15 Por tôpico Eduardo Henrique Leitner
hahaha, mas você usou calculadora né?!?!

orra, se você fez tudo aquilo na mão cara... eu te respeito! (ou não)

mas valeu ae

On Wed, Jul 16, 2003 at 01:36:59AM +0200, Fernando Henrique Ferraz Pereira da Rosa 
wrote:
  Incidentalmente há uns 2 anos também 'trombei' com esse problema e tive a
 paciência de resolver o sistema para achar a resposta. De qualquer forma
 postei uma mensagem para a lista e me mandaram uma sugestão mais simples:
  http://www.mat.puc-rio.br/~nicolau/olimp/obm-l.200107/msg4.html
 
 
 -- 
 []'s
 
 +++ GMX - Mail, Messaging  more  http://www.gmx.net +++
 
 Jetzt ein- oder umsteigen und USB-Speicheruhr als Prämie sichern!
 
 =
 Instruções para entrar na lista, sair da lista e usar a lista em
 http://www.mat.puc-rio.br/~nicolau/olimp/obm-l.html
 =
=
Instruções para entrar na lista, sair da lista e usar a lista em
http://www.mat.puc-rio.br/~nicolau/olimp/obm-l.html
=


[obm-l] Re: [obm-l] Re: [obm-l] Re: Como os Matemáticos Complicam II

2003-07-15 Por tôpico Nicolau C. Saldanha
On Wed, Jul 09, 2003 at 05:57:13PM -0300, J.Paulo roxer ´til the end wrote:
 Não está sendo muito produtiva esta lista pra mim.Entendo muito pouco.

Eu me arrependo profundamente de ter permitido que esta pessoa voltasse
a se inscrever na lista. Por favor vah embora e n~ao volte a escrever
para esta lista nunca mais, nem com este e-mail nem com nenhum outro.

Estou escrevendo da IMO. Os resultados devem sair dentro de algumas horas.

[]s, N.
=
Instruções para entrar na lista, sair da lista e usar a lista em
http://www.mat.puc-rio.br/~nicolau/olimp/obm-l.html
=


[obm-l]

2003-07-15 Por tôpico luiz-ernesto
Alguém me ajude com essa questão: 
Qual é o maior valor inteiro que não supera o número: 
( 2exp(2003)+3exp(2003)/(2exp(2001)+3exp(2001)) 

_
Voce quer um iGMail protegido contra vírus e spams?
Clique aqui: http://www.igmailseguro.ig.com.br
Ofertas imperdíveis! Link: http://www.americanas.com.br/ig/
Ofertas imperdíveis!

=
Instruções para entrar na lista, sair da lista e usar a lista em
http://www.mat.puc-rio.br/~nicolau/olimp/obm-l.html
=


Re: [obm-l]

2003-07-15 Por tôpico A. C. Morgado
( 2exp(2003)+3exp(2003)/(2exp(2001)+3exp(2001)) = 5exp(2003)/5exp(2001) = exp(2) 
aproximadamente
2,7^2 = 7,3
A resposta eh 7.


[EMAIL PROTECTED] wrote:

Alguém me ajude com essa questão: 
Qual é o maior valor inteiro que não supera o número: 
( 2exp(2003)+3exp(2003)/(2exp(2001)+3exp(2001)) 

_
Voce quer um iGMail protegido contra vírus e spams?
Clique aqui: http://www.igmailseguro.ig.com.br
Ofertas imperdíveis! Link: http://www.americanas.com.br/ig/
Ofertas imperdíveis!
=
Instruções para entrar na lista, sair da lista e usar a lista em
http://www.mat.puc-rio.br/~nicolau/olimp/obm-l.html
=
 

=
Instruções para entrar na lista, sair da lista e usar a lista em
http://www.mat.puc-rio.br/~nicolau/olimp/obm-l.html
=


Re: [obm-l]

2003-07-15 Por tôpico Frederico Reis Marques de Brito
Aexpressão é igual a 5exp(2003) / 5exp(2001)  = exp(2003)/exp(2001)= exp(2) 
. Desde que  e é aproximadamente igual a 2,7, decorre que a parte inteira de 
 e^2 = 7.  Logo, 7 é o maior inteiro que não supera o número dado pela 
expressão.


From: [EMAIL PROTECTED]
Reply-To: [EMAIL PROTECTED]
To: [EMAIL PROTECTED]
Subject: [obm-l] Date: Tue, 15 Jul 2003 23:55:56 -0300
Alguém me ajude com essa questão:
Qual é o maior valor inteiro que não supera o número:
( 2exp(2003)+3exp(2003)/(2exp(2001)+3exp(2001))
_
Voce quer um iGMail protegido contra vírus e spams?
Clique aqui: http://www.igmailseguro.ig.com.br
Ofertas imperdíveis! Link: http://www.americanas.com.br/ig/
Ofertas imperdíveis!
=
Instruções para entrar na lista, sair da lista e usar a lista em
http://www.mat.puc-rio.br/~nicolau/olimp/obm-l.html
=
_
MSN Hotmail, o maior webmail do Brasil.  http://www.hotmail.com
=
Instruções para entrar na lista, sair da lista e usar a lista em
http://www.mat.puc-rio.br/~nicolau/olimp/obm-l.html
=


Re: [obm-l]

2003-07-15 Por tôpico Eduardo Henrique Leitner
o q significa 5exp(2003)?

On Wed, Jul 16, 2003 at 12:46:20AM -0300, Frederico Reis Marques de Brito wrote:
 Aexpressão é igual a 5exp(2003) / 5exp(2001)  = exp(2003)/exp(2001)= exp(2) 
 . Desde que  e é aproximadamente igual a 2,7, decorre que a parte inteira 
 de e^2 = 7.  Logo, 7 é o maior inteiro que não supera o número dado pela 
 expressão.
 
 
 From: [EMAIL PROTECTED]
 Reply-To: [EMAIL PROTECTED]
 To: [EMAIL PROTECTED]
 Subject: [obm-l] Date: Tue, 15 Jul 2003 23:55:56 -0300
 
 Alguém me ajude com essa questão:
 Qual é o maior valor inteiro que não supera o número:
 ( 2exp(2003)+3exp(2003)/(2exp(2001)+3exp(2001))
 
 _
 Voce quer um iGMail protegido contra vírus e spams?
 Clique aqui: http://www.igmailseguro.ig.com.br
 Ofertas imperdíveis! Link: http://www.americanas.com.br/ig/
 Ofertas imperdíveis!
 
 =
 Instruções para entrar na lista, sair da lista e usar a lista em
 http://www.mat.puc-rio.br/~nicolau/olimp/obm-l.html
 =
 
 _
 MSN Hotmail, o maior webmail do Brasil.  http://www.hotmail.com
 
 =
 Instruções para entrar na lista, sair da lista e usar a lista em
 http://www.mat.puc-rio.br/~nicolau/olimp/obm-l.html
 =
=
Instruções para entrar na lista, sair da lista e usar a lista em
http://www.mat.puc-rio.br/~nicolau/olimp/obm-l.html
=


[obm-l] Conjuntos numeráveis e conjuntos não enumeráveis

2003-07-15 Por tôpico Artur Costa Steiner
Bom dia a todos,
Eu gostaria de levantar um assunto que há algum tempo me intriga. O fato
de um conjunto ser ou nao numeravel eh algo intrinseco ao conjunto ou
depende da topologia nele definida? Vou tentar explicar porque isso me
intriga. Para tanto, consideremos o conjunto R dos reais com a topologia
usual, definida pela metrica Euclidiana. Nesta situacao, sabemos que R
eh completo. Vamos agora analisar uma prova classica de que R nao eh
numeravel. Tomemos entao o intervalo I = [0,1] e seja X ={x_1,...
x_k...} uma enumeracao de elementos de I. Como todo elemento de R eh
ponto de acumulacao do mesmo, podemos escolher um subintervalo fechado
I_1 de I que nao contenha x_1. Da mesma forma, podemos escolher um
subintervalo fechado I_2 de I_1 que nao contenha x_2. Atraves de um
raciocinio indutivo, constatamos que este processo gera uma sequencia
{I_k} de subintervalos fechados de I tal que, para cada k, x_k nao
pertence a I_k. Logo, nenhum elemento da enumeracao X eh comum a todos
os intervalos I_k. Mas como R eh completo, existe, segundo um conhecido
teorema, um elemento x comum a todos os I_k que, consequentemente,
pertence a I mas nao estah englobado na enumeracao X. Isto nos mostra
que nenhuma enumeracao de elementos de I cobre a totalidade de I, do que
deduzimos que I e, portanto, o proprio R, nao sao numeraveis. 

Mas para que esta prova seja valida, precisamos saber previamente que R
eh completo e que todos seus elementos sao pontos de acumulacao do
mesmo. Tais condicoes dependem da topologia definida em R. Se, por
exemplo, tivermos R estruturado com a topologia definida pela metrica
discreta, entao a prova acima deixa de valer. Embora R continue sendo
completo (as sequencias de Cauchy passam a ser aquelas que se tornam
constantes a partir de algum k), nenhum elemento de R, na metrica
discreta, eh ponto de acumulacao do mesmo. Logo, os requisitos basicos
para a prova mencionada nao mais vigoram.

Na realidade, a prova que reproduzi para I eh um caso particular de uma
outra que diz que se um espaco topologico S eh compacto e nao contem
pontos isolados, entao S nao eh numeravel. O que acarreta que se um
espaco topologico contem um sub-espaco com as caracteristicas de S,
entao o espaco nao eh numeravel. Mas, os conceitos de conjunto compacto
e de pontos isolados dependem da topologia definida.

Uma outra prova da nao enumerabilidade de R eh a de Cantor, baseada em
expansoes decimais dos numeros reais. Mas esta prova tambem pressupoem R
com a topologia usual.

E eh isto que me intriga. Eh posivel provar que R nao eh numeravel sem
admitirmos alguma topologia nele definida? O fato de provarmos que um
conjunto eh ou nao numeravel numa topologia garante que tais condicoes
sao preservadas em qualquer topologia? 

Um abraco
Artur

=
Instruções para entrar na lista, sair da lista e usar a lista em
http://www.mat.puc-rio.br/~nicolau/olimp/obm-l.html
=


Re: [obm-l] polinomios

2003-07-15 Por tôpico Eduardo Casagrande Stabel
Oi chará!

O polinômio Q(x) = P(x) - 1 é de grau 5 e tem como raízes {1, 2, 3, 4, 5},
portanto Q(x) = A(x - 1)(x - 2)(x - 3)(x - 4)(x - 5), para algum A. Sabemos
que Q(6) = P(6) - 1 = - 1 = A*5!, logo A = -1/5!. O valor de P(0) = Q(0) + 1
= (-1)(-2)(-3)(-4)(-5)/(-5!) + 1 = 2. Portanto P(0) = 2. Acho que é isto.

Abraço!
Duda.


From: Eduardo Henrique Leitner [EMAIL PROTECTED]
 Fundamentos de Matemática Elementar, volume 6

 Gelson Iezzi

 145. Seja P(x) um polinômio de 5^o grau que satisfaz as condições:

 1 = P(1) = P(2) = P(3) = P(4) = P(5)
 e
 0 = P(6).

 Qual o valor de P(0)?


=
Instruções para entrar na lista, sair da lista e usar a lista em
http://www.mat.puc-rio.br/~nicolau/olimp/obm-l.html
=


[obm-l] Re: [obm-l] Conjuntos numeráveis e conjuntos não enumerávei

2003-07-15 Por tôpico Nicolau C. Saldanha
On Wed, Jul 16, 2003 at 02:24:02AM -0300, Artur Costa Steiner wrote:
 Bom dia a todos, Eu gostaria de levantar um assunto que há algum tempo me
 intriga. O fato de um conjunto ser ou nao numeravel eh algo intrinseco ao
 conjunto ou depende da topologia nele definida?

O conceito de cardinal 'e um dos poucos que est intrinseco ao conjunto, n~ao
depende de nenhum outro tipo de estrutura, seja topol'ogica ou qq outra.

  Vou tentar explicar porque isso me
  intriga. Para tanto, consideremos o
  conjunto R dos reais com a topologia
  usual, definida pela metrica Euclidiana.
  Nesta situacao, sabemos que R eh
  completo. Vamos agora analisar uma prova
  classica de que R nao eh numeravel.
  Tomemos entao o intervalo I = [0,1] e
  seja X ={x_1,...  x_k...} uma enumeracao
  de elementos de I. Como todo elemento de
  R eh ponto de acumulacao do mesmo,
  podemos escolher um subintervalo fechado
  I_1 de I que nao contenha x_1. Da mesma
  forma, podemos escolher um subintervalo
  fechado I_2 de I_1 que nao contenha x_2.
  Atraves de um raciocinio indutivo,
  constatamos que este processo gera uma
  sequencia {I_k} de subintervalos
  fechados de I tal que, para cada k, x_k
  nao pertence a I_k. Logo, nenhum
  elemento da enumeracao X eh comum a
  todos os intervalos I_k. Mas como R eh
  completo, existe, segundo um conhecido
  teorema, um elemento x comum a todos os
  I_k que, consequentemente, pertence a I
  mas nao estah englobado na enumeracao X.
  Isto nos mostra que nenhuma enumeracao
  de elementos de I cobre a totalidade de
  I, do que deduzimos que I e, portanto, o
  proprio R, nao sao numeraveis. 
 
 Mas para que esta prova seja valida, precisamos saber previamente que R eh
 completo e que todos seus elementos sao pontos de acumulacao do mesmo. Tais
 condicoes dependem da topologia definida em R.

Sim, a prova que voc^e acabou de dar depende da topologia de R.  Ali'as depende
tamb'em da ordem. Mas n~ao importa, R continua a ser n~ao enumer'avel mesmo sem
esta estrutura (a demonstra,c~ao pode n~ao funcionar, mas a conclus~ao ainda 'e
correta).
  Se, por exemplo,
  tivermos R
  estruturado com
  a topologia
  definida pela
  metrica
  discreta, entao
  a prova acima
  deixa de valer.
  Embora R
  continue sendo
  completo (as
  sequencias de
  Cauchy passam a
  ser aquelas que
  se tornam
  constantes a
  partir de algum
  k), nenhum
  elemento de R,
  na metrica
  discreta, eh
  ponto de
  acumulacao do
   

Re: [obm-l]

2003-07-15 Por tôpico Rafael Ando
5exp(2003) eh 5*e^2003, sendo exp(n) = e^n


From: Eduardo Henrique Leitner [EMAIL PROTECTED]
Reply-To: [EMAIL PROTECTED]
To: [EMAIL PROTECTED]
Subject: Re: [obm-l]
Date: Wed, 16 Jul 2003 01:35:43 -0300
o q significa 5exp(2003)?

On Wed, Jul 16, 2003 at 12:46:20AM -0300, Frederico Reis Marques de Brito 
wrote:
 Aexpressão é igual a 5exp(2003) / 5exp(2001)  = exp(2003)/exp(2001)= 
exp(2)
 . Desde que  e é aproximadamente igual a 2,7, decorre que a parte 
inteira
 de e^2 = 7.  Logo, 7 é o maior inteiro que não supera o número dado pela
 expressão.


 From: [EMAIL PROTECTED]
 Reply-To: [EMAIL PROTECTED]
 To: [EMAIL PROTECTED]
 Subject: [obm-l] Date: Tue, 15 Jul 2003 23:55:56 -0300
 
 Alguém me ajude com essa questão:
 Qual é o maior valor inteiro que não supera o número:
 ( 2exp(2003)+3exp(2003)/(2exp(2001)+3exp(2001))
 
 _
 Voce quer um iGMail protegido contra vírus e spams?
 Clique aqui: http://www.igmailseguro.ig.com.br
 Ofertas imperdíveis! Link: http://www.americanas.com.br/ig/
 Ofertas imperdíveis!
 
 
=
 Instruções para entrar na lista, sair da lista e usar a lista em
 http://www.mat.puc-rio.br/~nicolau/olimp/obm-l.html
 
=

 _
 MSN Hotmail, o maior webmail do Brasil.  http://www.hotmail.com

 
=
 Instruções para entrar na lista, sair da lista e usar a lista em
 http://www.mat.puc-rio.br/~nicolau/olimp/obm-l.html
 
=
=
Instruções para entrar na lista, sair da lista e usar a lista em
http://www.mat.puc-rio.br/~nicolau/olimp/obm-l.html
=
_
MSN Hotmail, o maior webmail do Brasil.  http://www.hotmail.com
=
Instruções para entrar na lista, sair da lista e usar a lista em
http://www.mat.puc-rio.br/~nicolau/olimp/obm-l.html
=